Continuity and Polar Coordinates

Click For Summary
The function f(x,y) = xy/sqrt(x^2+y^2) is analyzed for continuity at the origin using polar coordinates. Converting to polar coordinates results in f(r, θ) = r^2 sin(θ) cos(θ). The key point is that as r approaches 0, the limit of f(r, θ) does not depend on θ, indicating that the function approaches 0 uniformly from any direction. This contrasts with Cartesian coordinates, where limits can vary based on the path taken to the origin. Ultimately, proving that the limit equals f(0,0) confirms the continuity of the function at the origin.
schmidt7100
Messages
1
Reaction score
0

Homework Statement


Show that the function f(x,y)= xy/sqrt(x^2+y^2) is continuous at the origin using polar coordinates. f(x,y)=0 if (x,y)=(0,0)


Homework Equations


r=sqrt(x^2+y^2)
x=rcos(theta)
y=rsin(theta)


The Attempt at a Solution


So, converting this equation to polar coordinates, I get rsin(theta)cos(theta). However, after this I'm stumped as to how I prove that this is continuous at the origin.
 
Physics news on Phys.org
At the origin in polar coordinates (r,θ), shouldn't r=0 and θ=0 ?
 
Not exactly- at the origin r= 0 and \theta is undefined.

However, Schmidt7100, the point is that r alone measures the distance of a point from the origin- \theta is irrelevant. That means that, if \lim_{r\to 0}f(r,\theta) does not depend on \theta, then that is the limit of f(r, \theta) as (r, \theta) goes to the origin.

Note that is NOT the case for Cartesian coordinates. Since the distance from (x, y) to (0, 0) depends upon both x and y. I am sure you have seen examples of functions in (x, y) that give different limits as you go to (0, 0) along different paths.
 
You don't have to use polar co-ordinates, let
<br /> x_{n}=\frac{1}{n},\quad y_{n}=\frac{1}{n}<br />
Insert these into your equation for f and let n\rightarrow\infty, compute the limit and if it's the same as f(0,0) you've proved continuity.
 
Question: A clock's minute hand has length 4 and its hour hand has length 3. What is the distance between the tips at the moment when it is increasing most rapidly?(Putnam Exam Question) Answer: Making assumption that both the hands moves at constant angular velocities, the answer is ## \sqrt{7} .## But don't you think this assumption is somewhat doubtful and wrong?

Similar threads

  • · Replies 20 ·
Replies
20
Views
2K
  • · Replies 7 ·
Replies
7
Views
3K
  • · Replies 19 ·
Replies
19
Views
2K
  • · Replies 3 ·
Replies
3
Views
3K
  • · Replies 14 ·
Replies
14
Views
3K
  • · Replies 6 ·
Replies
6
Views
2K
  • · Replies 7 ·
Replies
7
Views
1K
Replies
3
Views
2K
Replies
3
Views
2K
  • · Replies 1 ·
Replies
1
Views
2K